Answer

问题及解答

证明: $\lim\limits_{n\rightarrow\infty}(1+\frac{1}{n})^n$ 极限存在.

Posted by haifeng on 2020-10-07 10:46:46 last update 2021-09-29 19:47:50 | Edit | Answers (3)

证明: $\lim\limits_{n\rightarrow\infty}(1+\frac{1}{n})^n$ 极限存在. 这里 $n\in\mathbb{Z}$.

 

记此极限为 $e$. 即

\[
\lim_{n\rightarrow\infty}(1+\frac{1}{n})^n=e
\]

一般的, 对于实数 $x$, 也有

\[
\lim_{x\rightarrow\infty}(1+\frac{1}{x})^x=e
\]

 


$f(x)=(1+\frac{1}{x})^x$ 这个函数, 如果视之为实函数, 则其定义域为 $(-\infty,-1)\cup(0,+\infty)$. 对于 $[-1,0]$ 中几乎所有的 $x$, $f(x)$ 都是(非实数的)复数. 不过可以证明下面两个结果.

\[
\lim_{x\rightarrow 0^+}(1+\frac{1}{x})^x=1.
\]

\[
\lim_{x\rightarrow -1^{-}}(1+\frac{1}{x})^x=+\infty.
\]

 

 

 

1

Posted by haifeng on 2020-10-07 13:42:18

我们首先证明极限 $\lim_{n\rightarrow+\infty}(1+\frac{1}{n})^n$ 存在.

令 $a_n=(1+\frac{1}{n})^n$, 我们证明数列 $\{a_n\}_{n=1}^{+\infty}$ 是严格单调递增数列, 并且是有界的.

Prop. $a_n < a_{n+1}$.

Pf. 这里 $a_n=(1+\frac{1}{n})^n$ 是二项式, 我们将之展开, 观察与 $a_{n+1}$ 展开后的关系.

\[
\begin{split}
a_n=(1+\frac{1}{n})^n&=\sum_{k=0}^{n}C_n^k\cdot 1^{n-k}\cdot(\frac{1}{n})^k\\
&=1+\sum_{k=1}^{n}\frac{n(n-1)\cdots(n-k+1)}{k!}\cdot\frac{1}{n^k}\\
&=1+\sum_{k=1}^{n}\frac{1}{k!}\cdot(1-\frac{1}{n})(1-\frac{2}{n})\cdots(1-\frac{k-1}{n})\\
\end{split}
\]

 

\[
\begin{split}
a_{n+1}=(1+\frac{1}{n+1})^{n+1}&=\sum_{k=0}^{n+1}C_{n+1}^k\cdot 1^{n+1-k}\cdot(\frac{1}{n+1})^k\\
&=1+\sum_{k=1}^{n+1}\frac{(n+1)(n+1-1)(n+1-2)\cdots(n+1-k+1)}{k!}\cdot\frac{1}{(n+1)^k}\\
&=1+\sum_{k=1}^{n+1}\frac{1}{k!}\cdot(1-\frac{1}{n+1})(1-\frac{2}{n+1})\cdots(1-\frac{k-1}{n+1})\\
\end{split}
\]

由于

\[
\sum_{k=1}^{n}\frac{1}{k!}\cdot(1-\frac{1}{n})(1-\frac{2}{n})\cdots(1-\frac{k-1}{n})<\sum_{k=1}^{n}\frac{1}{k!}\cdot(1-\frac{1}{n+1})(1-\frac{2}{n+1})\cdots(1-\frac{k-1}{n+1})
\]

故 $a_n < a_{n+1}$. 因此 $\{a_n\}_{n=1}^{+\infty}$ 是严格单调递增的数列.

 

另一方面, $\{a_n\}_{n=1}^{+\infty}$  是有界的. 事实上,

\[
\begin{split}
0 < a_n&=(1+\frac{1}{n})^n\\
&=\sum_{k=0}^{n}C_n^k\cdot 1^{n-k}\cdot(\frac{1}{n})^k\\
&=1+\sum_{k=1}^{n}\frac{n(n-1)\cdots(n-k+1)}{k!}\cdot\frac{1}{n^k}\\
&=1+\sum_{k=1}^{n}\frac{1}{k!}\cdot(1-\frac{1}{n})(1-\frac{2}{n})\cdots(1-\frac{k-1}{n})\\
& < 1+\sum_{k=1}^{n}\frac{1}{k!}\\
& < 1+1+\sum_{k=2}^{n}\frac{1}{k(k-1)}\\
&=2+\sum_{k=2}^{n}(\frac{1}{k-1}-\frac{1}{k})\\
&=2+\Bigl[(\frac{1}{1}-\frac{1}{2})+(\frac{1}{2}-\frac{1}{3})+\cdots+(\frac{1}{n-1}-\frac{1}{n})\Bigr]\\
&=2+1-\frac{1}{n}\\
& < 3
\end{split}
\]

因此 $\{a_n\}_{n=1}^{+\infty}$ 是有界的.

由定理, 该数列极限存在, 即 $\lim\limits_{n\rightarrow+\infty}a_n$ 存在, 我们将此极限记为 $e$.


当 $n\rightarrow-\infty$ 时, 令 $m=-n$, 从而

\[
\begin{split}
\lim_{n\rightarrow-\infty}(1+\frac{1}{n})^n&=\lim_{m\rightarrow+\infty}(1+\frac{1}{-m})^{-m}\\
&=\lim_{m\rightarrow+\infty}\frac{1}{(1-\frac{1}{m})^m}\\
&=\lim_{m\rightarrow+\infty}\frac{1}{(\frac{m-1}{m})^m}\\
&=\lim_{m\rightarrow+\infty}(\frac{m}{m-1})^m\\
&=\lim_{m\rightarrow+\infty}(1+\frac{1}{m-1})^m\\
&=\lim_{m\rightarrow+\infty}(1+\frac{1}{m-1})^{m-1}\cdot(1+\frac{1}{m-1})\\
&=e\cdot 1\\
&=e
\end{split}
\]

也就是说, 我们证明了

\[
\lim_{n\rightarrow\infty}(1+\frac{1}{n})^n=e.
\]

 

 

 

 


References:

梅加强 《数学分析》

2

Posted by haifeng on 2021-09-27 10:13:36

现在考虑一般情形, 即函数 $u(x)=(1+\frac{1}{x})^x$. 先考虑 $x > 0$.

这里使用 $[x]$ 表示不超过 $x$ 的最大整数, 于是 $[x]\leqslant x < [x]+1$.

我们的思路是将 $u(x)$ 控制在两个数列之间, 而这两个数列形如 $(1+\frac{1}{n})^n$,  从而使用夹逼准则证明其亦趋于 $e$.

\[
\begin{split}
&1+\frac{1}{[x]+1} < 1+\frac{1}{x} \leqslant 1+\frac{1}{[x]}\\
\Rightarrow\ &(1+\frac{1}{[x]+1})^{[x]} < (1+\frac{1}{x})^{[x]}\leqslant(1+\frac{1}{x})^x\leqslant(1+\frac{1}{[x]})^x < (1+\frac{1}{[x]})^{[x]+1}
\end{split}
\]

\[
(1+\frac{1}{[x]+1})^{[x]}=(1+\frac{1}{[x]+1})^{[x]+1}\cdot\frac{1}{1+\frac{1}{[x]+1}}\rightarrow e,\quad\text{当}\ x\rightarrow+\infty
\]

\[
(1+\frac{1}{[x]})^{[x]+1}=(1+\frac{1}{[x]})^{[x]}\cdot(1+\frac{1}{[x]})\rightarrow e,\quad\text{当}\ x\rightarrow+\infty
\]

因此, 

\[
\lim_{x\rightarrow+\infty}(1+\frac{1}{x})^x=e.
\]


对于 $x\rightarrow-\infty$ 的情形, 证明与之前负整数时类似.

 

3

Posted by haifeng on 2021-09-29 20:01:39

(1)

\[
\lim_{x\rightarrow 0^+}(1+\frac{1}{x})^x=\lim_{x\rightarrow 0^+}e^{x\ln(1+\frac{1}{x})}=e^{\lim_{x\rightarrow 0^+}x\ln(1+\frac{1}{x})}.
\]

令 $t=\frac{1}{x}$, 则

\[
\lim_{x\rightarrow 0^+}x\ln(1+\frac{1}{x})=\lim_{t\rightarrow +\infty}\frac{\ln(1+t)}{t}=0.
\]

因此,

\[
\lim_{x\rightarrow 0^+}(1+\frac{1}{x})^x=e^0=1.
\]


(2)

\[
\lim_{x\rightarrow (-1)^-}(1+\frac{1}{x})^x=\lim_{x\rightarrow (-1)^-}e^{x\ln(1+\frac{1}{x})}
\]

此时 $1+\frac{1}{x}\rightarrow 0^+$, 于是 $\ln(1+\frac{1}{x})\rightarrow -\infty$.

因此, 当 $x\rightarrow (-1)^-$ 时, $x\ln(1+\frac{1}{x})\rightarrow +\infty$, 从而有

\[
\lim_{x\rightarrow (-1)^-}(1+\frac{1}{x})^x=+\infty
\]